AHZolfaghari

Well-Known Member
ارسال ها
935
لایک ها
1,654
امتیاز
93
#21
پاسخ : ماراتن هندسه

میخواید یه راهنمایی بذارم براش . البته دوباره داره فعالیت بچه های سایت کم میشه ها . عطش خوابید فکر کنم . :(:(:(:(:(:(:(:149::149::149::149:
 

AHZolfaghari

Well-Known Member
ارسال ها
935
لایک ها
1,654
امتیاز
93
#22
پاسخ : ماراتن هندسه

میخواید یه راهنمایی بذارم براش . البته دوباره داره فعالیت بچه های سایت کم میشه ها . عطش خوابید فکر کنم . :(:(:(:(:(:(:(:149::149::149::149:
 

AHZolfaghari

Well-Known Member
ارسال ها
935
لایک ها
1,654
امتیاز
93
#23
پاسخ : ماراتن هندسه

چرا هیچ کی به تاپیک هندسه سر نمیزنه ؟؟ [MENTION=19860]REZA 73[/MENTION] [MENTION=17012]darya.f[/MENTION] [MENTION=18610]math1998[/MENTION]
 

math1998

New Member
ارسال ها
336
لایک ها
224
امتیاز
0
#24
پاسخ : ماراتن هندسه

بله درسته . سوال 4 ، سوال یک روز دوم المپیاد جهانی 2014 بود که البته آسون بود !

5
نقاط o ,h به ترتیب مراکز ارتفاعی و دایره محیطی مثلث abc هستند . و m ,n به ترتیب اوساط bh , ch هستند . و b'b قطر دایره است اگر honm محاطی باشد اثبات کنید b'n نصف ac است.
بهش میخوره دایره نه نقطه باشه .
 

AHZolfaghari

Well-Known Member
ارسال ها
935
لایک ها
1,654
امتیاز
93
#25
پاسخ : ماراتن هندسه

بهش میخوره دایره نه نقطه باشه .
سوال قشنگی هست و البته آسون نیست ( سوال tst ایران ) پس دنبال ایده های خوب باش !!!!!
 

mahmoud20ni

New Member
ارسال ها
18
لایک ها
13
امتیاز
3
#26
پاسخ : ماراتن هندسه

مرکز دایره محیطی چهارضلعی HMNO روی عمودمنصف HO و MN قرار دارد که هر دوی این عمودمنصف ها از مرکز دایره ی نه نقطه میگذره . پس یا مرکز دایره نه نقطه مرکز این دایره نیز میباشد که در این حالت به راحتی میتوان ثابت کرد که مثلث قائمه میباشد و ..... در غیر این صورت HMNO باید یک ذوزنقه ی متساوی الساقین باشد که در این حالت HO عمود بر خط گذرنده از روبرو قطری B و راس C خواهد بود که نتیجه میدهدHO عمود منصف این خط است پس مثلث b'HC متساوی الساقین میباشد اکنون از این موضوع که Hb' از وسط AC میگذرد استفاده کنید و حکم را نتیجه بگیرید
 

AHZolfaghari

Well-Known Member
ارسال ها
935
لایک ها
1,654
امتیاز
93
#27
پاسخ : ماراتن هندسه


6

در مثلث abc میدونیم که a برابر 60 درجه هستش . P روی bc هست بطوریکه 3bp = bc و i مرکز دایره محاطی است و f روی ab است بطوریکه if موازی ac است اثبات کنید زاویه bfp نصف زاویه b است
 

math1998

New Member
ارسال ها
336
لایک ها
224
امتیاز
0
#28
پاسخ : ماراتن هندسه

تموم ماراتن ها خوابیدن دیگ انشالله !!!!! تا چند ماه دیگه یادش بخیر پارسال دقیقه به دقیقه پست میذاشتن البته من نبودم اونموقع !!!!!
 

mahmoud20ni

New Member
ارسال ها
18
لایک ها
13
امتیاز
3
#29
پاسخ : ماراتن هندسه

برای حل سوال نقطه D را روی ضلع BC به گونه ای در نظر بگیرید که داشته باشیم
اکنون با قضیه تالس نشان میدهیم که
. پس باید نشان دهیم که
(که K نقطه برخورد FI با ضلع BC میباشد .) از طرفی :
و
اکنون حکم لازم برای استفاده از تالس را به صورت سینوسی در اورده و تمام زاویه ها را بر حسب c بنویسید و از پس از ساده سازی ها از اتحاد سینوسی زیر استفاده کنید :
 

m-saghaei

New Member
ارسال ها
338
لایک ها
258
امتیاز
0
#30
پاسخ : ماراتن هندسه

از نقطه
داخل مثلث
سه عمود بر اضلاع
فرود آورده و طولهای آنها را به ترتیب
مینامیم.اگر
طول ارتفاع های نظیر رئوس باشند ثابت کنید:
 

math1998

New Member
ارسال ها
336
لایک ها
224
امتیاز
0
#31
پاسخ : ماراتن هندسه

از نقطه
داخل مثلث
سه عمود بر اضلاع
فرود آورده و طولهای آنها را به ترتیب
مینامیم.اگر
طول ارتفاع های نظیر رئوس باشند ثابت کنید:
از نسبت مساحت ها به راحتی حل میشه .
 

darya.f

New Member
ارسال ها
182
لایک ها
114
امتیاز
0
#32
پاسخ : ماراتن هندسه

از نقطه
داخل مثلث
سه عمود بر اضلاع
فرود آورده و طولهای آنها را به ترتیب
مینامیم.اگر
طول ارتفاع های نظیر رئوس باشند ثابت کنید:
La x BC=2Sboc , ha x BC =2Sabc ...پس La/ha= Sboc/Sabc بقىرم همىنطورى مىنوىسىم ...
 
آخرین ویرایش توسط مدیر

darya.f

New Member
ارسال ها
182
لایک ها
114
امتیاز
0
#34
پاسخ : ماراتن هندسه

M,n روى AB,AC از مثلث ABC هستند که BC||MN , وBN,CM در P متقاطع اند.داىره محىطى مثلث هاى BMP و CNP ىکدىگر را در نقاط مجزاى P,Q,قطع کرده اند.ثابت کنىد BAQ=CAP
 

TheOverlord

New Member
ارسال ها
159
لایک ها
282
امتیاز
0
#35
پاسخ : ماراتن هندسه

BQN=BQP+PQN=PMA+PCN=CMA+MCA=180-A پس BQNA محاطي است. به طريق مشابه CQMA هم محاطي است. يعني BQA=BNA=PNA. حال ثابت ميكنيم كه AN/NP=AQ/QB تا در اين صورت طبق برابري يك زاويه و نسبت اضلاع مجاور ANP با AQB متشابه شود و در اين صورت PAN=BAQ كه همان حكم مساله است.
طبق تالس AN/AC=MN/BC=PN/PB پس AN/PN=AC/PB
پس كافيست بگوييم AC/PB=AQ/QB كه معادل است با AC/AQ=BP/BQ كه از تشابه CAQ و PBQ نتيجه ميشود، پس كافيست اين تشابه را ثابت كنيم. اما اين تشابه معادل است با اين كه QBP=QAC و BQP=AQC. اما BQP=PMA=CMA=CQA و QBP=QBN=QAN=QAC طبق چهارضلعي محاطي، پس دو برابري زاويه درستند، كه حكم مساله را نتيجه ميدهد.

سوال بعد: اگرI مركز دايره محاطي داخلي مثلث ABC باشد و Ia مركز دايره محاطي خارجي روبرو به راس A و Xa پاي عمود وارد از Ia بر BC ثابت كنيد IXa از وسط ارتفاع نظير راس A ميگذرد.
 

TheOverlord

New Member
ارسال ها
159
لایک ها
282
امتیاز
0
#36
پاسخ : ماراتن هندسه

راهنمايي: همساز بلد نيستيد منلائوس بزنيد!
 

Dadgarnia

New Member
ارسال ها
1,350
لایک ها
1,127
امتیاز
0
#37
پاسخ : ماراتن هندسه

BQN=BQP+PQN=PMA+PCN=CMA+MCA=180-A پس BQNA محاطي است. به طريق مشابه CQMA هم محاطي است. يعني BQA=BNA=PNA. حال ثابت ميكنيم كه AN/NP=AQ/QB تا در اين صورت طبق برابري يك زاويه و نسبت اضلاع مجاور ANP با AQB متشابه شود و در اين صورت PAN=BAQ كه همان حكم مساله است.
طبق تالس AN/AC=MN/BC=PN/PB پس AN/PN=AC/PB
پس كافيست بگوييم AC/PB=AQ/QB كه معادل است با AC/AQ=BP/BQ كه از تشابه CAQ و PBQ نتيجه ميشود، پس كافيست اين تشابه را ثابت كنيم. اما اين تشابه معادل است با اين كه QBP=QAC و BQP=AQC. اما BQP=PMA=CMA=CQA و QBP=QBN=QAN=QAC طبق چهارضلعي محاطي، پس دو برابري زاويه درستند، كه حكم مساله را نتيجه ميدهد.

سوال بعد: اگرI مركز دايره محاطي داخلي مثلث ABC باشد و Ia مركز دايره محاطي خارجي روبرو به راس A و Xa پاي عمود وارد از Ia بر BC ثابت كنيد IXa از وسط ارتفاع نظير راس A ميگذرد.
محل برخورد ارتفاع نظير راس A با BC را H و محل برخورد
با AH را M و محل برخورد AI با BC را D مي ناميم. با نوشتن قضيه ي منلائوس در مثلث AHD داريم:

پس براي اثبات حكم بايد نشان دهيم:

از طرف ديگر با توجه به تشابه مثلث هاي
داريم:

پس بايد نشان دهيم:

كه بنابر قضيه نيمساز صحيح است.
راه همسازي:
را ادامه مي دهيم تا IM را در E قطع كند. براي اثبات حكم كافي است نشان دهيم
. مي دانيم كه
و خطوط
در نقطه ي H همرسند پس چيزي كه مي خواستيم ثابت شد.
 
آخرین ویرایش توسط مدیر

TheOverlord

New Member
ارسال ها
159
لایک ها
282
امتیاز
0
#38
پاسخ : ماراتن هندسه

راه همسازي:
را ادامه مي دهيم تا im را در e قطع كند. براي اثبات حكم كافي است نشان دهيم . مي دانيم كه
و خطوط
در نقطه ي h همرسند پس چيزي كه مي خواستيم ثابت شد.
البته این راه خیلی درست نیست چرا که قضایای همسازی که استفاده کرده اید در صورت همخطی درستند، و شما نمیتوانید از درستیشان همخطی را نتیجه بگیرید. همچنین من متوجه استفاده شما از وسط بودن m نمیشوم.
 

Dadgarnia

New Member
ارسال ها
1,350
لایک ها
1,127
امتیاز
0
#39
پاسخ : ماراتن هندسه

البته این راه خیلی درست نیست چرا که قضایای همسازی که استفاده کرده اید در صورت همخطی درستند، و شما نمیتوانید از درستیشان همخطی را نتیجه بگیرید. همچنین من متوجه استفاده شما از وسط بودن m نمیشوم.
هم خطی
که از فرض های سواله و E رو هم که روی همین خط مشخص کردم و هم خطی
هم که بدیهیه و قسمت آخر سوالتون رو هم نفهمیدم منظورتون چیه.

---- دو نوشته به هم متصل شده است ----

سوال بعد:
در مثلث ABC،
است. نقطه متغیر D روی پاره خط BC را در نظر بگیرید. فرض کنید
مرکز دایره محیطی مثلث ABD و
مرکز دایره محیطی مثلث ACD باشد. محل تقاطع
و
را M و مرکز دایره محیطی مثلث
را N می نامیم. ثابت کنید خط MN از نقطه ثابتی می گذرد.
 

aras2213

New Member
ارسال ها
216
لایک ها
228
امتیاز
0
#40
پاسخ : ماراتن هندسه

هم خطی
که از فرض های سواله و E رو هم که روی همین خط مشخص کردم و هم خطی
هم که بدیهیه و قسمت آخر سوالتون رو هم نفهمیدم منظورتون چیه.

---- دو نوشته به هم متصل شده است ----

سوال بعد:
در مثلث ABC،
است. نقطه متغیر D روی پاره خط BC را در نظر بگیرید. فرض کنید
مرکز دایره محیطی مثلث ABD و
مرکز دایره محیطی مثلث ACD باشد. محل تقاطع
و
را M و مرکز دایره محیطی مثلث
را N می نامیم. ثابت کنید خط MN از نقطه ثابتی می گذرد.
AoPS Forum - MN passes through a constant point- Iran NMO 2005 - Problem2 • Art of Problem Solving
سوال بعد:
روی ضلع BC از متوازی الاضلاع ABCD که A حاده است نقطه T را طوری انتخاب میکنیم که ATD حاده الزاویه شود.اگر مراکز دایره های محیطیه ABT,DAT,CDT را به ترتیب X,Y,Z بنامیم نشان دهید مرکز ارتفاعیه XYZ روی AD قرار دارد.
 
بالا